Page 1 of 1

PT39 LR Q17

Posted: Fri Aug 24, 2012 8:39 pm
by Charlie.Home
I searched around for a good answer to this question, and couldn't find a clear one. Can anyone help explain it to me? Question is from PT39 LR Q17 (sorry I don't know which LR section it is from). I'll leave the correct answer off the post for those whom want to solve it, PM me if you want the answer.

Re: PT39 LR Q17

Posted: Fri Aug 24, 2012 9:34 pm
by CardozoLaw09
We're looking for a sufficient assumption that guarantees the conclusion stated in the stimulus. The conclusion of the stimulus is that systematic attempts by human beings to prevent or control forest fires are ill-advised and shortsighted. So answer choice B) completely fills in the gap left by the argument since it says that the ONLY legit reason to intervene is to protect forests and their ecosystems. The stimulus is basically saying that humans should leave forest fires alone and not try to prevent or control them because it's a natural phenomenon that facilitates positive effects for many forest ie) prevents an overabundance of insects. Further, the stim says that forest fires are required for many forests to flourish; however, if the forests and their ecosystems are in apparent danger of being destroyed then that would warrant human intervention to control or prevent the forest fires from doing so -- but that would be the ONLY reason to do so. Hope this helps!

Edit: You can't post full questions so make sure you edit that out